GMAT Critical Reasoning

Home > GMAT Test > GMAT Critical Reasoning Questions

Next steps

Source: OG

Level: 3

The proposal to hire 10 new police officers in Middletown is quite foolish. There is sufficient funding to pay the salaries of the new officers, but not the salaries of additional court and prison employees to process the increased caseload of arrests and convictions that new officers usually generate.

Which of the following, if true, will most seriously weaken the conclusion drawn above?

  • A Studies have shown that an increase in a city's police force does not necessarily reduce crime.
  • B When one major city increased its police force by 19 percent last year, there were 40 percent more arrests and 13 percent more convictions.
  • C If funding for the new police officers' salaries is approved, support for other city services will have to be reduced during the next fiscal year.
  • D In most United States cities, not all arrests result in convictions, and not all convictions result in prison terms.
  • E Middletown's ratio of police officers to citizens has reached a level at which an increase in the number of officers will have a deterrent effect on crime.

Show Answer

Previous       Next